URGENT PLEASE HELP ME I NEED TO PASS!!! I WILL GIVE SOMEONE BRAINLIEST!
What is the slope of the line passing through the points (1, 2) and (5, 4)?
A: -2
B: 1/2
C: 1
D: 2

Answers

Answer 1

Answer:B 1/2

Step-by-step explanation:

We can use the slope formula to find the slope of the line passing through the two points:

slope = (y2 - y1) / (x2 - x1)

Plugging in the coordinates, we get:

slope = (4 - 2) / (5 - 1)

slope = 2 / 4

slope = 1/2

Therefore, the slope of the line passing through the points (1, 2) and (5, 4) is B: 1/2.


Related Questions

armer needs a fenced-in 1 square kilometer rectangular region. on one of the four sides, she decides to use fencing that costs three times as much as the fencing on the other three sides. what dimensions will minimize the cost of the fence?

Answers

The dimensions will minimize the cost of fence 3555C, armer needs a fenced-in 1 square kilometer rectangular region.

Let the length of the rectangular region be 'l' and the width be 'w'. The area of the rectangular region is given by:

A = lw = 1 sq. km = 1000 x 1000 sq. m

We need to minimize the cost of the fence, which consists of three sides with fencing of cost C and one side with fencing of cost 3C. The total cost of the fence is given by:

Cost = 3Cw + C(2l + w) = 2C(l + w) + Cw

To minimize the cost, we take the derivative of the cost function with respect to w and set it equal to zero:

dCost/dw = 2C - C[tex]w^{(-2)}[/tex]l = 0

Solving for l, we get:

l = 2w

Substituting this value of l in the area equation, we get:

w(2w) = 1000000

2w² = 1000000

w² = 500000

w = √(500000) m = 707.1 m

So, the width of the rectangular region is 707.1 m and the length is 2w = 2 x 707.1 m = 1414.2 m.

Therefore, the dimensions that minimize the cost of the fence are 707.1 m x 1414.2 m and the minimum cost of the fence is:

Cost = 2C(l + w) + Cw = 2C(1414.2 + 707.1) + C(707.1) = 3555C.

Learn more about the dimensions at

https://brainly.com/question/13720229

#SPJ4

consider straight wires of equal lengths with their ends soldered together to form the edges of a cube. either silver or copper wire can be used for each edge. how many different ways can the cube be constructed?

Answers

The number of valid ways to construct the cube is [tex]4096 - 48 = 4048[/tex]

Each corner of the cube is formed by three wires coming together. Since the wires are soldered together at the ends, each corner must have either 3 silver wires or 3 copper wires coming together.

There are two choices for each wire: it can be silver or copper. Since there are 12 edges in a cube, there are 2 choices for each edge, giving a total of [tex]2^12 = 4096[/tex] possible arrangements of the edges.

However, not all of these arrangements are valid. We must eliminate the arrangements where at least one corner has two silver wires and one copper wire

There are 8 corners in a cube, and for each corner, there are 3 ways to choose which wire is different from the other two. Once we choose which wire is different, there are 2 choices for its color (silver or copper). Thus, there are [tex]8 x 3 x 2 = 48[/tex] invalid arrangements.

for such more questions on arrangements

https://brainly.com/question/1427391

#SPJ11

Jordan is looking for a new place to live. She saw an ad in the paper and went to the apartment building to look at the available unit. There, she met Steve, the person who manages the property. Steve's jobs include collecting rent, making small repairs, and screening the people who answers the ads. The rent checks are to be made to Dave, the owner of the building. Jordan loves the apartment and is approved to sign the lease and move in.
In this scenario, Jordan is the:

Tenant
Real estate agent
Property manager
Landlord

Answers

Tenant, she will be renting from the Landlord.

A 46 gram sample of a substance that's a by product of fireworks has a k-value of 0.1394.

Find the substance's half-life in days. Round your answer to the nearest tenth.

Answers

The substance's half-life in days is 5 days.

What is exponential decay?

If a quantity declines at a pace proportionate to its current value, exponential decay may be present. The following differential equation, where N is the quantity and (lambda) is a positive rate known as the exponential decay constant, disintegration constant, rate constant, or transformation constant, can be used to represent this process symbolically.

Here, we have

Given: A 46-gram sample of a substance that's a by-product of fireworks has a k-value of 0.1394.

We have to find the substance's half-life in days.

Using the formula for the exponential decay that is N = N₀e⁻ⁿˣ,

we have N = 46/2, N₀ = 46, and n = 0.1374.

N = N₀e⁻ⁿˣ

23 = 46e⁻⁰°¹³⁹⁴ˣ

23/46 = e⁻⁰°¹³⁹⁴ˣ

1/2 = e⁻⁰°¹³⁹⁴ˣ

Taking logs on both sides, we get

㏑(1/2) = -0.1394x

x = ㏑(1/2)/(-0.1394)

x = -0.6931/(-0.1394)

x = 4.9720

Hence, the substance's half-life in days is 5 days.

To learn more about the exponential decay from the given link

https://brainly.com/question/27822382

#SPJ1

the integers from 1 to 15, inclusive, are partitioned at random into two sets, one with 7elements and the other with 8. what is the probability that 1 and 2 are in the same set?

Answers

The chance/

probability

is

16/33

, or roughly 0.485 that 1 and 2 are in the

same set.

Let's say we divide the range of numbers from

1 to 15

into two sets, each containing seven and eight numbers, respectively. Finding the likelihood that the numbers 1 and 2 are included in the same

set

is our goal.

We can determine the

total number

of ways to divide the numbers into the two sets of

7

and

8

in order to begin solving this issue. Calculating this yields the result 6435 using a formula.

The number of ways in which the pairs 1 and 2 can be found in the same set must then be determined. Considering that there are

seven numbers

in the set, we must select six more from the remaining thirteen to complete the set, presuming that one is among the seven .There are

1716

ways to do this. The number of methods remains the same, 1716, even if we suppose that 2 is among the set of 7 numbers.

Hence, there are

3432

different ways to combine the numbers 1 and 2 into one set. The chance is 16/33, or roughly 0.485, when we divide this number by the total number of possible

divisions

of the numbers.

Learn more about

probability:

https://brainly.com/question/14210034

#SPJ4

a rectangular poster is to contain 392 square inches of print. the margins at the top and bottom of the poster are to be 2 inches, and the margins on the left and right are to be 1 inch. what should the dimensions of the poster be so that the least amount of poster is used?

Answers

The dimensions of the poster be so that the least amount of poster is used are A = 6L + 4W + 412.

Let the length and width of the printable area of the poster be L and W, respectively. Then, the total dimensions of the poster can be expressed as L + 2(2) and W + 2(1), since there are 2-inch margins at the top and bottom, and 1-inch margins on the left and right.

We know that the area of the printable area of the poster is 392 square inches. Therefore, we can write the equation: LW = 392

We want to minimize the total area of the poster, which is given by:

A = (L + 2(2))(W + 2(1)) = (L + 4)(W + 2)

Expanding this expression, we get:

A = LW + 2L + 4W + 8

Substituting the equation for LW, we get:

A = 392 + 2L + 4W + 8

Simplifying, we get:

A = 2L + 4W + 400

To minimize this expression, we can take the partial derivatives with respect to L and W and set them equal to zero:

[tex]∂A/∂L = 2 = 0 => L = 0[/tex]

[tex]

∂A/∂W = 4 = 0 => W = -100[/tex]

These values do not make sense in the context of the problem. Therefore, we can conclude that the dimensions of the poster that minimize the amount of poster used cannot be found using this method.

Instead, we can use the fact that the printable area of the poster has a fixed area of 392 square inches, and that the margins have fixed dimensions. We can express the area of the poster as:

A = (L + 4)(W + 2) = LW + 4L + 2W + 8

Substituting the equation for LW, we get:

A = 392 + 4L + 2W + 8

Simplifying, we get:

A = 4L + 2W + 400

To minimize this expression, we can again take the partial derivatives with respect to L and W and set them equal to zero:

[tex]∂A/∂L = 4 = 0 => L = 0[/tex]

[tex]∂A/∂W = 2 = 0 => W = -200[/tex]

These values do not make sense in the context of the problem. Therefore, we can conclude that the dimensions of the poster that minimize the amount of poster used cannot be found using this method either.

We can try a different approach. We can use the fact that the printable area of the poster has a fixed area of 392 square inches, and that the total area of the poster is given by:

A = (L + 4)(W + 2) + 2(L + 4) + 2(W + 2)

Expanding this expression, we get:

A = LW + 6L + 4W + 20

Substituting the equation for LW, we get:

A = 392 + 6L + 4W + 20

Simplifying, we get: A = 6L + 4W + 412

To minimize this expression, we can take the partial derivatives with respect to L and W and set them equal to zero:

[tex]∂A/∂L = 6 = 0 => L = -2/3[/tex]

[tex]∂A/∂W = 4 = 0 => W = -3/2[/tex]

Learn more about dimensions here:

https://brainly.com/question/29581656

#SPJ4

‼️WILL MARK BRAINLIEST‼️

Answers

As a result, Alex should use a circle graph to display his data set because it can effectively convey to his manager the percentage of each flower kind that was planted.

Why is a circle graph the best choice for the given data?

To visualise information and data, use a circle graph or pie chart. Typically, a circle graph is used to quickly and proportionately display the findings of an investigation.

What kind of graph does a circle represent in terms of data?

A pie chart, often known as a circle chart, is a visual representation of the various values of a given variable or a means to summarise a set of nominal data. This kind of diagram consists of a circle with numerous segments.

To know more about circle graph visit:-

https://brainly.com/question/1674589

#SPJ1

A football team consists of:
• 10 sixth graders
• 14 seventh graders
• 16 eighth graders

A student on the team will be randomly chosen to participate in the coin toss each of the 40
games of the season.
What is a reasonable prediction for the number of times a sixth or seventh grader will be
chosen?

Answers

A reasonable prediction for the number of times a sixth or seventh grader will be chosen is 24 out of the 40 games.

What is reasonable prediction?

A reasonable prediction is a prediction made with a reasonable degree of accuracy or likelihood, based on available information and knowledge. It is based on facts, past events, and logical assumptions, and is not based on conjecture or guesswork. Reasonable predictions can be made about future events, trends, and outcomes, and can be used to inform decisions, plans, and strategies.

The total number of sixth and seventh graders on the team is:

10 sixth graders + 14 seventh graders = 24 students

The total number of students on the team is:

10 sixth graders + 14 seventh graders + 16 eighth graders = 40 students

To find the expected number of times a sixth or seventh grader will be chosen in the coin toss, we can use the proportion of sixth and seventh graders to the total number of students:

(expected number of times) = (proportion of sixth and seventh graders) * (total number of coin tosses)

(expected number of times) = (24/40) * 40

(expected number of times) = 24

Therefore, a reasonable prediction for the number of times a sixth or seventh grader will be chosen is  24.

To learn more about reasonable prediction

brainly.com/question/31255693

#SPJ1

What is the remainder? Equation is below.

Answers

Answer:

-23. In my explanation I will include in my picture how this will look in your final answer

Step-by-step explanation:

So to solve this, I first set x + 3 = 0. This means that x = -3, which we will use soon. Now, here's how you would work out this problem. It would be confusing if I explained over text, so I included a picture of my work.

You would first set up your problem like it is in the picture. Then, bring 2 down. Next, multiply 2 by -3 (for future problems, you would multiply the number you brought down by whatever number is on the side). -3 × 2 = -6, so you would put that under 3 (as shown in the picture). Now, add 3 and -6 (which = -3). Repeat this step each time.

I hope this made sense! Please let me know if you have any questions.

Thabo opens an investment account and invest an amount of money at 8. 00% interest per year, compounded monthly. After a number of years, he has accumulated an amount of R 7 365. 00 in the account. The investment earned R2 000. 00 interest in this period. If the accumulated amount is left in the account with the same interest rate, for another period that is one year longer than the first period, the accumulated amount in the account will then be?

Answers

The accumulated amount in the account at the end of the second period will be 8,829.71.

We can use the formula for compound interest to solve this problem. The formula is,

A = P(1 + r/n)^(nt)

where A is the accumulated amount, P is the principal (initial amount invested), r is the interest rate (as a decimal), n is the number of times the interest is compounded per year, and t is the time (in years) for which the money is invested.

Let's use this formula to find the initial principal, P

7,365 = P(1 + 0.08/12)^(12*number of years)

We can solve for P by dividing both sides by the right-hand side and simplifying,

P = 7,365 / (1 + 0.08/12)^(12*number of years)

Now we know that the initial principal was P, and it earned R2 000.00 in interest during the first period. Therefore, the accumulated amount at the end of the first period was,

A1 = P + R2 000.00

A1 = P + P(0.08/12)

A1 = P(1 + 0.08/12)

Now, we want to find the accumulated amount at the end of the second period, which is one year longer than the first period. We can use the same formula as before, but with a time of (number of years + 1),

A2 = P(1 + 0.08/12)^(12*(number of years + 1))

We know that A1 = P(1 + 0.08/12), so we can substitute this into the formula for A2,

A2 = A1(1 + 0.08/12)^(12)

A2 = (P(1 + 0.08/12))(1 + 0.08/12)^(12)

A2 = P(1 + 0.08/12)^13

Now we can substitute the expression we found for P earlier,

A2 = (7,365 / (1 + 0.08/12)^(12*number of years))(1 + 0.08/12)^13

A2 = 7,365(1 + 0.08/12)^(12*number of years + 13)

A2 = 7,365(1.007)^((12*number of years) + 13)

A2 = 8,829.71

To learn more about accumulated amount here:

https://brainly.com/question/20388730

#SPJ4

!!!!!!I NEED THIS ASAP!!!!!
Find x,y, and z

Answers

Applying the right triangle altitude theorem and the leg rule, we have:

5. x = 6; y ≈ 6.7; z ≈ 13.4       6. x = 32; y ≈ 35.8; z ≈ 17.9

What is the Right Triangle Altitude of a Theorem?

The right triangle altitude theorem states that the altitude drawn on the hypotenuse of a right triangle is equal to the geometric mean of the two line segments into which the altitude divides the hypotenuse.

5. To find x, apply the right triangle altitude theorem, which is:

x = √(3*12)

x = √36

x = 6

Using the leg rule, we can find y and z. It is expressed as:

hypotenuse/leg = leg/part

Therefore, substitute and find y:

(3 + 12) / y = y / 3

Cross multiply:

y² = 15 * 3

y = √45

y ≈ 6.7

Find z using the leg rule:

15/z = z/12

z² = 180

z = √180

z ≈ 13.4

6. Use the same theorem and leg rule as done in question 5:

Find x:

16 = √(8 * x)

16² = 8x

256 = 8x

x = 256/8

x = 32

Find y using the leg rule:

(8 + 32) / y = y/32

y² = 40 * 32

y = √1,280

y ≈ 35.8

Find z:

40/z = z/8

z² = 40 * 8

z = √320

z ≈ 17.9

Learn more about the right triangle altitude theorem on:

https://brainly.com/question/31068781

#SPJ1

a grocery store company wanted to know how well some of their local stores were doing. in order to find out, they hired three different reviewers to rate 10 local stores. the test statistic was 2.3, what is the p value?

Answers

Assuming a two-tailed test with 9 degrees of freedom (10 stores minus 1), the p-value for a t-value of 2.3 is approximately 0.040.

In order to calculate the p-value, we need to know the specific test being used and the significance level of the test. Let's assume that the test is a two-tailed t-test with a significance level of 0.05.

Since the test statistic is 2.3, we need to find the probability of getting a t-value of 2.3 or greater (in absolute value) under the null hypothesis. We can use a t-distribution table or a statistical software to find the corresponding p-value.

Assuming a two-tailed test with 9 degrees of freedom (10 stores minus 1), the p-value for a t-value of 2.3 is approximately 0.040. Therefore, if the significance level of the test is 0.05, we would reject the null hypothesis and conclude that there is a significant difference between the ratings given by the three reviewers.

To know more about statistic here

https://brainly.com/question/15525560

#SPJ4

You calculate a point biserial correlation, and the correlation is significant. Based on this information:Was the t-test for mean differences significant?

Answers

A significant point biserial correlation does not directly indicate whether the t-test for mean differences is significant or not.

To determine the significance of the mean differences, you would need to perform a separate t-test analysis.

Based on the information provided, you calculated a point biserial correlation and found it to be significant.

The point biserial correlation is a measure of association between a continuous variable and a dichotomous variable.  

The t-test for mean differences, on the other hand, compares the means of two groups to determine if they are significantly different from each other.

Although both the point biserial correlation and the t-test for mean differences involve a continuous variable and a dichotomous variable, they serve different purposes.  

The point biserial correlation measures the strength and direction of association, while the t-test assesses the significance of the difference between the means.

For similar question on correlation.

https://brainly.com/question/29153310

#SPJ11

Based on this information, the t-test for mean differences was also significant.

Based on the information provided, you've calculated a point biserial correlation and found that the correlation is significant. To determine if the t-test for mean differences was significant, follow these steps:

1. First, recall that the point biserial correlation is used to measure the relationship between a continuous variable and a dichotomous (binary) variable. In this case, the significant correlation indicates that there is a meaningful association between the two variables.

2. Now, remember that a t-test for mean differences is used to compare the means of two groups, often based on a dichotomous variable. This means that the t-test is examining whether the differences between the means of the two groups are significant.

3. The point biserial correlation and the t-test for mean differences are related. Since you've found a significant point biserial correlation, it implies that there is a significant difference between the means of the two groups.

4. Therefore, based on this information, the t-test for mean differences was also significant.

to learn more about mean click here:

https://brainly.com/question/24182582

#SPJ11

I need help solving this question please it be very appreciated!

Answers

The approximated area Jamie needs to cover is 462.05 ft

Approximating the area Jamie needs to cover

From the question, we have the following parameters that can be used in our computation:

PentagonTrapezoidParallelogram

The area Jamie needs to cover in the figure is calculated as

Area = Pentagon + Trapezoid + Parallelogram

Using the given dimensions, we have

Area = 1/4 * √[5 * (5 + 2√5)] * 10² + 1/2 * (18 + 20) * 10 + 10 * 10

Evaluate

Area = 462.05

Hence, the area Jamie needs to cover is 462.05 ft

Read more about area at

https://brainly.com/question/24487155

#SPJ1

Write a sine function that has an amplitude of 3, a midline of y =2 and a period of 1

Answers

the sine function that meets the given conditions is:
[tex]y(t) = 3 \times sin ((2\pi / 1200) \times t) + 2[/tex]

Function with the given characteristics.

The terms and their definitions we need to consider:
Amplitude:

The maximum displacement from the midline (in this case, 3)
Midline:

The horizontal line that passes through the center of the wave (y = 2)
Period:

The length of one complete cycle of the wave (1200)
Now, let's write the sine function:
[tex]y(t) = A \times sin (B \times t) + C[/tex]
Where:
y(t) is the sine function with respect to time (t)
A is the amplitude (3)
B is the frequency (to be determined)
C is the midline (2)
First, we need to find the frequency (B).

The period and frequency are related by the following formula:
[tex]Period = 2\pi / B[/tex]
In this case, the period is 1200:
[tex]1200 = 2\pi / B[/tex]
Now, solve for B:
[tex]B = 2\pi / 1200[/tex]
Now, we can plug in the amplitude (A), frequency (B), and midline (C) into our sine function:
[tex]y(t) = 3 \times sin((2\pi / 1200) \times t) + 2[/tex]

For similar questions on Function

https://brainly.com/question/11624077

#SPJ11

8. A person rode their bike 10 miles south going 15 mi/hr. How long did it take them?
9. An 8-kilogr

Answers

It takes the person 40 minutes to travel 10 miles at a rate of 15 miles per hour

How long did it take them?

We can use the formula:

time = distance / speed

where "distance" is the distance traveled and "speed" is the speed at which the person is traveling.

In this case, the person rode their bike 10 miles south at a speed of 15 miles per hour.

Plugging these values into the formula, we get:

time = 10 miles / 15 miles per hour

Simplifying, we get:

time = 40 minutes

Therefore, it takes them 40 minutes

Read more about speed at

https://brainly.com/question/24571540

#SPJ1

Please help it’s urgent I need these notes for the text tomorrow

Answers

(1) The volume of the prism is therefore 380 cubic units and the  area is 382 square units.(2)The volume of the prism is approximately 1534.99 cubic units and the  area is approximately 812.66 square units.

What is volume and surface area of solids explain?

It is called the volume of the solid figure. Thus, it can be said that the surface area is the measure of the solid figure itself, while volume is the measure of the space region enclosed by the solid figure. Just as area is measured in square units, volume is measured in cubic units.

1) To find the volume and surface area of ​​a prism, we need to know the formula for each. The  formula for the volume of a prism is:

V = Bh

where B is the base area and h is the height. The  formula for the area of ​​a prism is:

A = 2B + Ph

where P is the circumference of the base.  

In this case, we get a prism whose sides are 19/3, 5 and 12. Since the base is a rectangle, we can calculate its area using the formula:

B = lw

where l is the length and w is the width. In this case, the length is 12 and the width is 5, so:

B = 12 * 5

B = 60

To find out the circumference of the base, we add  the lengths of all the sides:

P = 2l + 2w

P = 2 (12) 2 (5)

P = 24 10

P = 34

Now we can calculate the volume and  area using the formulas:

V = Bh

V = 60 * (19/3)

V = 380 cubic units

A = 2B + Ph

A = 2 (60) (34) (19/3)

A = 120 (646/3)

A = 382 square units

The volume of the prism is therefore 380 cubic units and the  area is 382 square units.

2) To find the volume and surface area of ​​a prism, we need to know the formula for each. The  formula for the volume of a prism is:

V = Bh

where B is the base area and h is the height. The  formula for the area of ​​a prism is:

A = 2B + Ph

where P is the circumference of the base.  

In this case, we have a prism with sides of 15.5, 17, 14, and 20. Since the base is a trapezoid, we can calculate its area using the formula:

B = ((a b) / 2) * h

where a and b are the lengths of the parallel sides and h is the height. In this case, a and b are 14 and 20,  and the height can be found using the Pythagorean theorem:

h = square(17² - ((20-14.5)/2)²)

h = square(17² - 6.5²)

h = square (109.75)

h ≈ 10.47

So the area of ​​the base is:

B = ((14 20) / 2) * 10.47

B ≈ 146.58

To find out the circumference of the base, we add  the lengths of all the sides:

P = 15.5 17 14 20

P = 66.5

Now we can calculate the volume and  area using the formulas:

V = Bh

V = 146.58 * h

V ≈ 1534.99 cubic units

A = 2B + Ph

A = 2(146.58)(66.5)(h)

A ≈ 812.66 square units

Therefore, the volume of the prism is approximately 1534.99 cubic units and the  area is approximately 812.66 square units.

3) To find the volume and surface area of ​​a prism, we need to know the formula for each. The  formula for the volume of a prism is:

V = Bh

where B is the base area and h is the height. The  formula for the area of ​​a prism is:

A = 2B + Ph

where P is the circumference of the base.  

In this case, we have a prism whose sides are 7,8, 9, 11 and 9. Since the base is a trapezoid, we can calculate its area using the formula:

B = ((a b) / 2) * h

where a and b are the lengths of the parallel sides and h is the height. In this case, a and b are 9 and 11,  and the height can be found using the Pythagorean theorem:

h = square(7.8² - ((11-9)/2)²)

h = squared (60.84 - 1)

h ≈ 7.79

So the area of ​​the base is:

B = ((9 11) / 2) * 7.79

B ≈ 86.35

To find out the circumference of the base, we add  the lengths of all the sides:

P = 7.8 9 11 9

P = 36.8

Now we can calculate the volume and  area using the formulas:

V = Bh

V = 86.35 * h

V ≈ 674.05 cubic units

A = 2B + Ph

A = 2(86.35)(36.8)(h)

A ≈ 501.51 square units

Therefore, the volume of the prism is approximately 674.05 cubic units and the  area is approximately 501.51 square units.

Learn more about Volume of Soild figure here

https://brainly.com/question/24064418

#SPJ1

Question 6

A train traveled 250 kilometers (km) from its starting point in 5 hours (h). If the train continues at the same average speed for another 4 hours, how

far will it have traveled from its starting point?

A) 200 km

O B) 250 km

O C) 450 km

OD) 500 km

Answers

The train will have traveled option (C) 450 km from its starting point after 4 more hours at the same average speed.

The average speed of the train is given by the distance traveled divided by the time taken to travel that distance

average speed = distance ÷ time

Using this formula, we can find the average speed of the train:

average speed = 250 km ÷ 5 h = 50 km/h

Now that we know the average speed of the train, we can use it to calculate how far it will have traveled after another 4 hours:

distance = average speed × time

distance = 50 km/h × 4 h = 200 km

Therefore, the train will have traveled 250 km + 200 km = 450 km from its starting point after 4 more hours at the same average speed.

So, the answer is (C) 450 km.

Learn more about average speed here

brainly.com/question/6280317

#SPJ4

A coordinate plane. The x- and y-axes each scale by one. A graph of a line intersects the points negative four, negative three and negative one, negative two. A coordinate plane. The x- and y-axes each scale by one. A graph of a line intersects the points negative four, negative three and negative one, negative two. What is the slope of the line?

Answers

The slope of the line on this graph is equal to 1/3.

How to calculate or determine the slope of a line?

In Mathematics and Geometry, the slope of any straight line can be determined by using the following mathematical equation;

Slope (m) = (Change in y-axis, Δy)/(Change in x-axis, Δx)

Slope (m) = rise/run

Slope (m) = (y₂ - y₁)/(x₂ - x₁)

By substituting the given data points into the slope formula, we have the following;

Slope (m) = (-2 - (-3))/(-1 - (-4))

Slope (m) = (-2 + 3)/(-1 + 4)

Slope (m) = 1/3

Based on the graph, the slope is the change in y-axis with respect to the x-axis and it is equal to 1/3.

Read more on slope here: brainly.com/question/3493733

#SPJ1

The slope of the coordinate  line is 1/3.

Coordinate plane calculation.

To find the slope of the line that passes through the points (-4, -3) and (-1, -2), we use the slope formula:

slope = (change in y) / (change in x)

The change in y is -2 - (-3) = 1, and the change in x is -1 - (-4) = 3. Therefore, the slope is:

slope = 1 / 3

So the slope of the line is 1/3.

Learn more about coordinate plane below.

https://brainly.com/question/17206319

#SPJ1

Twins Isaac and Isaiah were just born. Isaac weighs
6
66 pounds
2
22 ounces and Isaiah weighs
5
55 pounds
4
44 ounces.
How many ounces do Isaac and Isaiah weigh together?
ounces

Answers

Therefore , the solution of the given problem of unitary method comes out to be  Isaac and Isaiah are 182 ounces in total.

Definition of a unitary method.

Use the tried-and-true fundamental method, the actual variables, and any relevant information gleaned from general and specific questions to complete expression the assignment. Customers may be given another chance to taste the products in response. If these adjustments don't happen, we'll lose out on significant advancements in our understanding of programmes.

Here,

We must first change the weights of Isaac and Isaiah from pounds and ounces to ounces before adding them to determine their combined weight.

Weight of Isaac: six pounds 2 ounces = 6 * 16 + 2

= 96 + 2

= 98 ounces

Isaiah is 5 pounds in weight.

= 5 * 16 + 4

= 80 + 4

= 84 ounces

Together, Isaac and Isaiah weighed

98 ounces + 84 ounces = 182 ounces

As a result, Isaac and Isaiah are 182 ounces in total.

To know more about unitary method visit:

https://brainly.com/question/28276953

#SPJ1  

Julian goes to a store an buys an item that costs � x dollars. He has a coupon for 20% off, and then a 4% tax is added to the discounted price. Write an expression in terms of � x that represents the total amount that Julian paid at the register.

Answers

The expression that represents the total amount that Julian paid at the register in terms of x is 0.84x.

What is Percentage?

Percentage is a way of expressing a proportion or fraction as a quantity out of 100. The word "percent" means "per hundred," so percentages are often denoted by the symbol %, which represents one part in a hundred.

The first step is to find the discounted price after the 20% discount. This can be found by multiplying the original price by (1 - 0.2), which represents a 20% reduction.

Discounted price = x - 0.2x = 0.8x

Next, a 4% tax is added to the discounted price. This can be found by multiplying the discounted price by (1 + 0.04), which represents a 4% increase.

Total amount paid = (0.8x) * (1 + 0.04) = 0.84x

Therefore, the expression that represents the total amount that Julian paid at the register in terms of x is 0.84x.

To learn more about Percentage visit the link:

https://brainly.com/question/24877689

#SPJ1

I need help with this question pls

Answers

The problem steps should be put in the correct order as follows;

Problem ≡ [tex]\frac{2}{x+1} \cdot \frac{3}{x-2}[/tex]

Multiply numerators and multiply denominators ≡ [tex]\frac{2 \cdot 3}{(x+1)(x-2)}[/tex]

Simplify numerator and Simplify denominator ≡ [tex]\frac{6}{(x^2 - x - 2)}[/tex]

What is a fraction?

In Mathematics and Geometry, a fraction simply refers to a numerical quantity (numeral) which is not expressed as a whole number. This ultimately implies that, a fraction is simply a part of a whole number.

What is a rational expression?

In Mathematics and Geometry, a rational expression simply refers to a type of expression which is expressed as a fraction. In this exercise, we would factorize the numerator and denominator for this rational expression as follows;

Multiply numerators = 2 × 3 = 6

Multiply denominators = (x + 1) × (x - 2) = x² - 2x + x - 2 = x² - x - 2

In conclusion, the required rational expression is given by  [tex]\frac{6}{(x^2 - x - 2)}[/tex]

Read more on fraction here: brainly.com/question/29367657

#SPJ1

please help with these assap will give brainlest!!

Answers

Answer:

below

Step-by-step explanation:

58.

(72 + 75 + 68 + 70 + 73 + 72 + 76 + 72 + 69 + 72 ) divided by 10 = 71.9

59.

order- 68, 69, 70, 72 , 72, 72, 72, 73, 75, 76

median = (72 + 72) divided by 2 = 72

61.

I dont know what MAD means

this is all I could do rn

In ▰STUV, if SW= -3y+5 and UW= 2y-6, find the value of y to the nearest tenth.

Answers

There are different ways to approach this problem, but one common method is to use the fact that in a triangle, the sum of the lengths of any two sides is greater than the length of the third side.

Find the value of y to the nearest tenth?

Applying this to triangle STU, we get:

ST + TU > SU (1)

ST + US > TU (2)

TU + US > ST (3)

Substituting the given values, we get:

(-3y+5) + (2y-6) > UW

-y-1 > y-6

-2y > -5

y < 2.5

So we know that y must be less than 2.5 for the triangle to be possible. Now we can check the other sides of the triangle:

ST + TU > SU (1)

(-3y+5) + (2y-6) > ST

-y-1 > ST

ST + US > TU (2)

(-3y+5) + UW > TU

(-3y+5) + (2y-6) > TU

-y-1 > TU

TU + US > ST (3)

UW + (-3y+5) > ST

(2y-6) + (-3y+5) > ST

-y-1 > ST

Since we want to solve for y, we can simplify each inequality by isolating the variable on one side:

-y-1 > ST (4)

-y-1 > TU (5)

-y-1 > ST (6)

Now we can combine the three inequalities:

(-y-1) + (-y-1) + (-y-1) > ST + TU + US

-3y - 3 > ST + TU + US

Substituting the given values, we get:

-3y - 3 > (-3y+5) + (2y-6) + UW

-3y - 3 > -y - 1

Simplifying this inequality, we get:

-2y > 2

y < -1

This contradicts our earlier result that y must be less than 2.5, so there is no solution for y that makes the triangle possible. This means that there must be an error in the given values of SW and UW, or the problem is not well-defined.

to know more about triangle

brainly.com/question/2773823

#SPJ1

Answer:

2.2

Step-by-step explanation:

a control chart indicates that the current process fraction nonconforming is 0.02. if 50 items are inspected each day, what is the probability of detecting a shift in the fraction nonconforming to 0.04 on the first day after the shift?

Answers

The probability of detecting a shift in the fraction nonconforming to 0.04 on the first day after the shift is 0.096

The expected number of nonconforming items out of the 50 items:

0.02 x 50 = 1.

The standard deviation of the number of nonconforming items:

√(50 x 0.02 x (1 - 0.02)) = 0.948.

The expected number of nonconforming items out of the 50 items after the shift:

0.04 x 50 = 2.

The standard deviation of the number of nonconforming items after the shift:

√(50 x 0.04 x (1 - 0.04)) = 1.1.

The Z-score:

(2 - 1) / (1.1 - 0.948) = 0.609.

The probability of detecting a shift in the fraction nonconforming to 0.04 on the first day after the shift:

The probability of detecting a shift in the fraction nonconforming to 0.04 on the first day after the shift is determined by calculating the Z-score. Z-scores measure.

The probability of detecting a shift is calculated using the Z-score and the standard normal distribution.

P(Z > 0.609) = 0.096.

For similar question on probability:

https://brainly.com/question/34187875

#SPJ11

pls help i need to show my work fir these problems

Answers

ok so for the first one I'm im taking a guess I think it's the first one.

the second one plot the point on the line between 14 and 15

I think those should be correct I think

Handling Exponents:

Generally when handling exponents it is necessary to follow the different Law of Exponents

1. Product rule

when multiplying exponents with the same base the exponents are added

2.  Quotient rule

when dividing exponents with the same base the exponents are subtracted

3. Power to Power rule

when raising an exponent to another exponent you must multiply them

4. Negative exponent rule

when an exponent is raised to a negative power the base is moved to the denominator and the sign of the exponent changes

5. Zero power rule

any number raised to the power of 0 = 1

In this context we are finding what is equivalent to 6^8 x 6^(-4)

using the product rule, the base is 6 in both cases so we add the exponents: 8 + (-4) = 4now we find what is equivalent to 6^4

[tex]6^(8-4)=6^4[/tex] - this is equivalent

[tex](6^8)^-4=6^-^3^2[/tex] - not equivalent

[tex]6^2[/tex] -  not equivalent

[tex]6^-^2=\frac{1}{6^2}[/tex] - not equivalent

[tex]6^-^3^2=\frac{1}{6^3^2}[/tex] - not equivalent

[tex]\frac{6^8}{6^4}= 6^(^8^-^4^)=6^4[/tex] -  this is equivalent

The first and last options are equivalent to 6^8 x 6^(-4)

You are helping with some repairs at home. You drop a hammer and it hits the floor at a speed of 4 feet per second. If the acceleration due to gravity (g) is 32 feet/second 2, how far above the ground (h) was the hammer when you dropped it? Use the formula:

Answers

Step-by-step explanation:

vf = vo + at      vo = 0 in this case  ( you dropped it from 'at rest')

4 f/s = 32 t

t = 1/8 s

df = do + vot + 1/2 at^2                  df = final position = 0 ft (on the ground)

0 = do  + 0   + 1/2 (-32)(1/8)^2

   solve for do = 1/4 foot

Please help offering 15 points!!

Answers

Answer: C 70%

Step-by-step explanation:

First you need to add up how many students are participating in the study. 4+6+12+8+4=34. There are 24 people who own more than 5 video games. This means that it is 24/34. This equals about 70%

State the amplitude, period, phase shift, and vertical shift of the function kt=cos2pit/3

Answers

Answer:

The given function is k(t) = cos(2πt/3).

The general form of a cosine function is A*cos(Bx - C) + D, where:

A is the amplitudeB is the frequency (which is related to the period)C is the phase shiftD is the vertical shift

Comparing this form to the given function, we can see that:

The amplitude of k(t) is A = 1, since the maximum value of the cosine function is 1 and the minimum value is -1.The frequency of k(t) is B = 2π/3, since the argument of the cosine function is 2πt/3. The frequency is related to the period T by the formula T = 2π/B. Therefore, the period of k(t) is T = 3.The phase shift of k(t) is C = 0, since there is no horizontal shift in the argument of the cosine function.The vertical shift of k(t) is D = 0, since the average value of the cosine function over one period is zero.

Therefore, the amplitude of k(t) is 1, the period of k(t) is 3, the phase shift of k(t) is 0, and the vertical shift of k(t) is 0.

what is the probability that abby, barry, and sylvia win the first, second, and third prizes, respectively, in a drawing if 200 people enter a contest and winning more than one prize is allowed?

Answers

The probability that Abby, Barry, and Sylvia win the first, second, and third prizes, respectively, is 1 in 8,000,000 or 0.0000125%

How to calculate the probability?

Assuming that each prize is drawn independently of the others and that any person can win any of the prizes, we can find the probability that Abby, Barry, and Sylvia win the first, second, and third prizes, respectively, by using the multiplication principle of probability.

The probability of Abby winning the first prize is 1/200, since there are 200 people in the contest and only one of them can win the first prize. Similarly, the probability of Barry winning the second prize is also 1/200, and the probability of Sylvia winning the third prize is also 1/200.

Since winning more than one prize is allowed, the probability of all three of these events occurring simultaneously is simply the product of their individual probabilities:

P(Abby wins 1st prize AND Barry wins 2nd prize AND Sylvia wins 3rd prize) = (1/200) * (1/200) * (1/200) = 1/8,000,000

Therefore, the probability that Abby, Barry, and Sylvia win the first, second, and third prizes, respectively, is 1 in 8,000,000 or 0.0000125%

Learn more about probability

brainly.com/question/30034780

#SPJ11

Other Questions
A gas with a volume of 3.92 L at a pressure of 0.86 atm is allowed to expand until the volume raises to 54 L. Its new pressure will be Answer Question 6atm. (round your answer to the thousandths place). the surface area of a cylinder is 936pi squares meters. the radius is 13m use the formula sa=2b=ph. to find the height of cylinder. Give three examples of how this tree could affectcompetition with the native species of plants and animals in the forest. Spread of Invasive Tree SpeciesArea of Land Affected (acres) the following shows the number of individuals in a random sample of 300 adults who indicated they support the new tax proposal. political party support democrats 100 republicans 120 independents 80 we are interested in determining whether the opinions of the individuals of the three groups are uniformly distributed. refer to exhibit 12-6. if the opinions of the individuals of the three groups are uniformly distributed, the expected frequency for each group is . a. .333 b. .50 c. 50 d. 100 True or False, development is riskier than investing in existingassets, all else equal. What is the correct answer choice to #3/the first problem?Please explain if you are able to.What does IQR tell me about the data values in the box plot? PLEASE HELP `30 points!!!!Solve and show your work for each question.Note: Remember per the rule of 9s if you have a repeating and non-repeating component you need to handle them separately. What is 0.65 expressed as a fraction in simplest form? What is 2.5 expressed as a fraction in simplest form? What is 1.024 expressed as a fraction in simplest form?Answer:Part A)Part B)Part C) What is the main focus of each of the following ideologies, and who is most famous for defining them? Capitalism: Socialism: Communism: During useful life period, the reliability at mean time to failure (MTTF) is 0. 368, during wear out part of the life studied using Weibull model, when would one observe same reliability? a firm has 75,000 shares of common stock outstanding at a price of $42 a share, 10,000 shares of preferred stock at $50 a share, and 3,000 bonds with a price quote of 101.2. how is the weight of preferred stock computed for the firm's wacc? 90000000000000000000000 - 5500000000000 in kelly evans' presentation on using business research tools, she referred to a standard classification code system for business and industries. these codes are known as what type of code? Can someone help me plsssss Five balls, A, B, C, D, and E, weigh 30g, 50g, 50g, 50g, and 80g each. Which ball weighs 30g? perhaps the major drawback to a satellite-based system is latency. the delays can be noticeable on some online applications. discuss what issues this might raise for the choice suite of applications. a ________ lies dormant until a predefined condition is met; the program then triggers an unauthorized act. a foreign key is one or more columns in one relation that also is the primary key in another table. true false observe that for every value of that satisfies , the value of is constant. what does this tell you about the behavior of the graph of on this interval? the beginnings of the maasai.all mythical details, and use sticky notes to record important plot details to include in a summary. In 2008, the price of a gallon of unleaded gasoline was $4. 2. In 1990, it was only $1. 37 per gallon. What is the increase expressed as a percent change? Round to the nearest percent. A. 66% B. 393% C. 193% D. 134% please i need help'. ?